M VIII Flashcards

1
Q

A man committed a serious offence when he was age 15. At the time, the man was living in a state which had signed the European Convention on Human Rights (‘ECHR’). The man served a sentence of 10 years and was released. Two years later, the state decided to deport the man back to his home country. The man argued that this would be in breach of his right to family life as all his family now live in the state. He has had an unblemished record since his first offence, and he has a job. The state argued that the seriousness of the crime means that the young man remains a danger to the public. The young man was unsuccessful in the highest court of the state. He brings his case to the European Court of Human Rights (‘ECtHR’).

Which of the following statements represents the man’s best argument to convince the ECtHR that he should be allowed to stay?

That deporting him is not reasonable and does not support a reasonable aim, and that there are more reasonable ways of achieving the same aim.

That deporting him is not fair and does not support a fair aim, and that there are fairer ways of achieving the same aim.

That deporting him is not an effective way of achieving the state’s aim and there are more restrictive ways of achieving the same aim.

That deporting him does not rationally support the state’s aim and there are more restrictive ways of achieving the same aim.

That deporting him does not rationally support the state’s aim, and there are less onerous ways of achieving the same aim.

A

E) The man’s best argument is that deporting him does not rationally support the state’s aim and there are less onerous ways of achieving that aim. The man claims that his deportation would violate his right to family life, which is a qualified right. Interference with qualified rights must be proportional. This choice properly describes the proportionality test. (A) and (B) are incorrect because those choices reference reasonableness and fairness, respectively, which are not terms used in the proportionality test. (C) and (D) are incorrect because they reference “more” restrictive means rather than less restrictive means.

How well did you know this?
1
Not at all
2
3
4
5
Perfectly
2
Q

A scouting association appointed a new volunteer scout leader, after ensuring that she had the required qualifications and experience. The association entrusted the leader with the task of taking a group of young adults on an overnight camping trip. Due to the leader’s carelessness in failing to inspect the camp area, the group set up their tents perilously close to an unmarked ravine. During the night, one of the scouts fell into the ravine and was injured. The injured woman brings an action for negligence against the scouting association.

Can the scouting association be liable for the negligent actions of the scout leader?

Yes, because the scouting association could be vicariously liable for the actions of the scout leader if there is a relationship akin to employment.

No, because the scout leader is a volunteer and she is not employed by the scouting association

Yes, because the scouting association is responsible for using reasonable care in selecting scout leaders.

No, because the scouting association has no duty of care towards the injured woman.

Yes, because the scouting association appointed the scout leader and the scout leader is therefore employed by it.

A

(A) The scouting association can be liable. Although the scout leader is a volunteer and therefore not employed by the scouting association, the claimant may be able to establish a relationship akin to employment sufficient to give rise to vicarious liability on the part of the scouting association. The court may find that such relationship exists because the scout leader’s tort (negligence) was committed as a result of an activity undertaken on behalf of the association, the scout leader’s activity was an integral part of the association’s activities, and the association created the risk of the scout leader’s tort by assigning leadership of the camping trip to her. (B) is incorrect in a case such as this where a relationship akin to employment is created. (C) is incorrect. It is true that the scouting association may itself owe a duty to use reasonable care in selecting scout leaders. However, on the facts, this duty has been complied with because the association made certain that the leader had the required qualifications and experience. (D) is incorrect. The scouting association may be vicariously liable, as explained above. Also, it is not correct to say that the scouting association did not itself owe a duty of care to the injured woman. It is likely to have owed her a duty to use reasonable care in selection of the scout leader (which, on the facts, it did). (E) is incorrect because the scout leader is a volunteer. The fact that she was appointed by the scouting association is not sufficient to make her an employee of the association.

How well did you know this?
1
Not at all
2
3
4
5
Perfectly
3
Q

An employee of a chemical manufacturing company has approached a solicitor, asking her to act for him in a personal injury claim involving a chemical spillage at the company. Until two years ago, other solicitors at the firm acted for the same company, advising on a variety of issues, including employment, environmental, and financial issues. The company ended its retainer when it decided to appoint different solicitors as legal advisers.

Which of the following best describes how the solicitor should proceed in the circumstances?

Act for the employee, because the solicitor has easy access to information about the company’s processes.

Refuse to act for the employee, but contact the company and offer to act for it in the case.

Act for the employee, because different solicitors at the firm acted for the company.

Act for the employee, because the company ended its retainer two years ago.

Act for the employee, provided appropriate safeguards are in place regarding confidential information.

A

(E) The fact that the company is a former client does not end the duty of confidentiality. There may well be confidential information, for instance, in relation to environmental matters. (D) is therefore incorrect. The firm cannot act where it holds confidential information material to the employee’s case, unless either (1) measures are taken to ensure there is no real risk of disclosure, or (2) the former client consents. (A) is incorrect because the information is confidential to the company. (B) is incorrect because it is improper to contact another solicitor’s client in such circumstances. (C) is incorrect because the entire firm owes a duty of confidentiality to the company. Therefore, if it holds confidential information that is material to the matter, it cannot act for the employee unless safeguards are in place or the company consents.

How well did you know this?
1
Not at all
2
3
4
5
Perfectly
4
Q

A solicitor is acting for a client in relation to the purchase of a substantial shareholding in a company. The transaction is progressing well but the client is due to go on holiday and will be out of contact for a few days. The client gives the solicitor authority to sign any necessary documents on the client’s behalf in his absence. The solicitor duly signs the share purchase contract. Neither the solicitor nor her firm is authorised by the Financial Conduct Authority to carry on a “regulated activity” as defined in the Financial Services and Markets Act 2000 (‘FSMA’) and related secondary legislation.

Which of the following best describes the activity specified in the Financial Services and Markets Act 2000 (Regulated Activities) Order 2001, which the solicitor carried out in signing the share purchase contract?

Advising

Arranging

Dealing as Agent

Managing

Safeguarding

A

C) Under the general prohibition of FSMA, a solicitor may not carry on regulated activity in the UK unless they are authorised or exempt from authorisation. A regulated activity is defined as an activity specified in the Regulated Activities Order relating to an investment specified in the Order, which is carried out in the course of business, and to which no exclusions apply. The specified activities might be remembered with the mnemonic device ‘ADAMS’. Activities = Advising, Dealing as an agent, Arranging, Managing, and Safeguarding. Dealing as agent is buying, selling, subscribing for, or underwriting investments as agent for a client. Here, the solicitor contracted for the purchase of the shares as agent on the client’s behalf, which is the specified activity of dealing as agent. (A) is incorrect because, in signing the share purchase agreement, the solicitor was not advising, that is, giving advice to the client which the client would then act upon. (B) is incorrect because the solicitor was not making arrangements, that is, arranging for another person to buy, sell, subscribe for, or underwrite a particular investment. As explained above, in signing the share purchase agreement, the solicitor contracted for the purchase of the shares herself on behalf of the client. (D) is incorrect because the solicitor was not managing, that is, managing assets belonging to another person in circumstances which involve the exercise of discretion. The solicitor was not exercising discretion as to which shares to buy, but rather implementing an agreed share purchase transaction. (E) is incorrect because the solicitor was not safeguarding, that is, administering or safeguarding (looking after) the shares. This specified activity is more relevant to probate and trust work

How well did you know this?
1
Not at all
2
3
4
5
Perfectly
5
Q

A husband runs a printing business. His wife asks him to print a number of brochures for her pet grooming business. The husband agrees to print the brochures for £100. After the brochures have been printed, the wife refuses to pay.

What is the position under contract law?

The wife has no legal obligation to pay. As they are married, this constitutes a domestic and social arrangement where there is a presumption that the parties do not intend to be legally bound.

The wife has a legal obligation to pay. As they are married, this constitutes a domestic and social arrangement where there is a presumption that the parties intend to be legally bound.

The wife has a legal obligation to pay. Even though they are married, they both were acting in a commercial context.

The wife has no legal obligation to pay as she agreed to do so before the brochures were printed, so there is no consideration for the parties’ promises.

The wife has a legal obligation to pay her husband for any costs incurred in the printing so he is not out of pocket, but nothing more.

A

C) Even though the parties are husband and wife, and there is a presumption that they do not intend to be legally bound, that presumption would be rebutted by the fact that they are both acting in a business capacity. (A) is therefore incorrect for the same reason. Whilst marriage raises a presumption that there is no legal intent, it can be rebutted by evidence to the contrary, as in this scenario. (B) is incorrect because in a domestic and social arrangement there is no presumption that the parties intended to be legally bound. That is the position in a commercial context. (D) is incorrect because the timing of the wife’s agreement is irrelevant. The promise to pay was consideration for both promises and is executory (a promise to do something in the future). (E) is incorrect because in this context the law does not distinguish between costs incurred and the contractual obligation to pay. Either the wife is legally obliged to pay or she is not, and in this case it would appear that she Is.

How well did you know this?
1
Not at all
2
3
4
5
Perfectly
6
Q

A company owns and runs a fitness gym. On several occasions over the past few months, company employees have caught a youth entering the gym without permission to use the fitness equipment. They warned him against entering without paying, but he did it again the following week. He began to use one of the fitness machines which then collapsed onto him, causing him a serious injury and smashing his expensive mobile phone. The evidence later showed that the machine had a worn safety bolt. The company running the gym had been aware of this serious and dangerous defect but had not found time to have it repaired.

In a claim by the youth against the company for damages for his personal injury and property loss, which of the following statements is correct?

The youth is not likely to have any successful claim against the company because he entered the premises and used the machine without permission, so the company did not owe him a duty of care.

The youth is not likely to have any successful claim against the company because, while the company owed a duty of care to the youth, it was not in breach of that duty.

The youth is not likely to have any successful claim against the company because, while the company owed a duty of care to the youth to take reasonable care to see that he does not suffer injury to his person, it was not in breach of that duty, and because it did not owe him a duty of care as to his property.

The youth is likely to have a successful claim against the company for all of his damages because the company breached its duty to take reasonable care to see that he does not suffer injury to his person or property.

The youth is likely to have a successful claim against the company for his personal injury, but not for his mobile phone, because the company breached its duty to take reasonable care to see that he does not suffer death or personal injury.

A

(E) The youth can recover damages for his personal injury but not for his mobile phone. The youth suffered an injury caused by the state of the premises (the defective fitness machine). The company is the occupier of the premises. The youth is not a lawful visitor; he entered the premises as a trespasser. In these circumstances, the duty owed is governed by the Occupiers’ Liability Act 1984. An occupier owes a duty to a trespasser provided the following are satisfied: the occupier is aware of the danger (the company knew of the defect); the company was aware that a trespasser may come into the vicinity (it was aware of the youth’s previous trespasses); and it would be reasonable to expect the occupier to offer protection (the defect was serious and dangerous). The duty is to take reasonable care to see that the trespasser does not suffer injury. The duty does not cover damage to property. On the facts, the duty appears to have been breached because the occupier was aware of the dangerous defect in the machine but had not found time to have it repaired. Causation of damage is also satisfied. Therefore, the youth should have a claim in respect of his personal injury but not for the damage to his mobile phone. (A) is not correct because the company did owe the youth a duty to take reasonable care to see that he does not suffer injury to his person. (B) is not correct. The company does appear to be in breach of the duty owed to the youth, as discussed above. (C) is not correct in two respects. First, the youth is not likely to have a successful claim in respect of his mobile phone because he was a trespasser. Second, the company did breach its duty as to the youth’s personal injury, as discussed above. (D) is not correct. The company’s duty to see that the youth does not suffer injury applies only to personal injury, not to property damage.

How well did you know this?
1
Not at all
2
3
4
5
Perfectly
7
Q

Recently, an American lawyer retained a client who trades with businesses in Wales. The client is concerned over the fact that the Welsh Parliament has passed an Act that amends certain retained EU law. The UK government has not passed any regulations barring the Welsh Parliament from modifying this particular retained EU law. Moreover, the retained EU law concerns a matter falling within the Welsh Parliament’s legislative competence.

Which of the following best explains the legal position?

The Act is valid because the Welsh Parliament is not bound to comply with this retained EU law.

The Act is not valid because the Welsh Parliament is bound to comply with this retained EU law.

The Act is not valid because it has not been approved by the Supreme Court.

The Act is valid because it has been approved by the UK Parliament.

The Act is valid because the devolved legislatures may legislate contrary to any retained EU law.

A

(A) The Act is valid because the Welsh Parliament is not bound to comply with this retained EU law. With the end of the transition period (in December 2020), the devolved legislatures are now free to legislate contrary to retained EU law falling within their legislative competence, unless excepted by regulations passed by the UK government under the European Union (Withdrawal) Act 2018. Because the UK government has not passed any such regulations, and this retained EU law falls within the Welsh Parliament’s legislative competence, the Welsh Parliament was free here to legislate contrary to the retained EU law. (B) is incorrect because legislation passed by the Welsh Parliament after the end of the transition period does not need to comply with this retained EU law. (C) and (D) are incorrect because those procedures do not exist. (E) is incorrect because it is too broad. The devolved legislatures may legislate contrary to retained EU law, assuming the retained EU law is within the devolved institutions’ legislative competence and is not excepted by regulations passed by the UK government.

How well did you know this?
1
Not at all
2
3
4
5
Perfectly
8
Q

A legal research intern wants to explore the appeals process and so asks a senior judge to explain the direction that appeals can be made.

Which of the following is not a direction by which an appeal can be made?

The Crown Court to the Court of Appeal
The Court of Appeal to the Supreme Court
The High Court to the Supreme Court
The High Court to the Court of Appeal
The Crown Court to the High Court
A

(E) Appeals from the Crown Court will be heard in the Criminal Division of the Court of Appeal, as can be seen in (A). As in (B) the Court of Appeal decision can be appealled to the Supreme Court, and as in (D) the High Court’s decision can be appealed to the civil division of the Court of Appeal. (C) the High Court’s decision can be appealed straight through to the Supreme Court if the matter is of public significance.

How well did you know this?
1
Not at all
2
3
4
5
Perfectly
9
Q

A solicitor is acting for a claimant in a claim against their former financial adviser. The parties complied with the pre-action protocol but could not settle the claim. As a result, the claimant issued proceedings. The claimant’s solicitor then received notice that the claim was proposed to be allocated to the multi-track. The claimant’s solicitor is confident of success but is aware of potential weaknesses in the claim and of the escalating costs now that court proceedings are underway. Therefore, the solicitor would like to explore settlement at this stage.

Which of the following describes the most appropriate course of action for the solicitor to adopt with a view to achieving settlement at this stage of the claim?

The claimant’s solicitor should make an immediate application for summary judgment.

The claimant’s solicitor should request further information to narrow the issues stated in the defence.

The claimant’s solicitor should ask the defendant for an interim payment.

The claimant’s solicitor should write an open letter to the defendant asking them to admit liability for the advice.

The claimant’s solicitor should ask the court to order a stay for alternative dispute resolution to enable potential settlement to be explored.

A

(E) The solicitor should ask for a stay for alternative dispute resolution (for example, mediation or negotiated settlement). This is likely to be granted by the District Judge and it will give the parties the opportunity to consider the strengths of their respective cases and actively engage to see if settlement can be achieved at this point. (A) is incorrect because an application for summary judgment asks the court to enter judgment for the applicant without proceeding to trial because evidence shows the other party has no real prospect of success. As the facts indicate the solicitor is aware of weaknesses in the claim and we are at an early stage of the proceeding, it is unlikely an application for summary judgment will succeed. (B) is incorrect because, although a request for information can narrow issues, such a request is not a mechanism to promote settlement. (C) is incorrect because the grant of an interim payment is not a settlement device either; it is merely an application asking the court to order the other party to pay something before the case is settled. Such an order can be ‘without prejudice’ - meaning the court could rule in the paying party’s favour later. (D) is incorrect because the defence already disputes liability. The defendant is unlikely to respond to a request to admit with anything other than a reiteration of the denial set out in the defence.

How well did you know this?
1
Not at all
2
3
4
5
Perfectly
10
Q

A consumer wanted to invest her modest savings. She could not afford to pay for financial advice. However, she obtained free advice from an investment advisor who has the practice of providing the first half hour of consultation free of charge. The consumer invested her savings in reliance on the advice. Unfortunately, the investment fell in value and she lost all her money. An independent review later showed that her loss was caused by the negligence of the investment advisor, whose advice had been totally inappropriate.

In a claim in tort by the consumer against the advisor, which of the following best describes the duty of the advisor?

The advisor did not owe a duty of care to the consumer because the advice was given for free.

The advisor did not owe a duty of care to the consumer because she suffered pure economic loss.

The advisor did owe a duty of care to the consumer because he was in business as an investment advisor.

The advisor did owe a duty of care to the consumer because he knew that she was relying solely on his advice to invest her savings and it was reasonable for her to do so.

The advisor did not owe a duty of care to the consumer because it was not reasonable for her to rely on his advice without further independent enquiry.

A

(D) The advisor did owe a duty of care to the consumer. The consumer would consider a claim in the tort of negligence and would need to establish that the advisor owed her a duty of care. The consumer has suffered pure economic loss. The advisor would only owe a duty of care in respect of such loss if there were a special relationship involving an assumption of responsibility by the advisor and reasonable reliance by the consumer. This choice correctly recognises such a relationship. The advisor knew the purpose of the advice, that it would be communicated to the consumer, and that she would rely on it without independent enquiry. She did rely on it and it was reasonable for her to do so. (A) is not correct. The fact that the advice was given for free does not prevent the duty of care from arising. On these facts, a duty would be owed. (B) is not correct. It is true that the consumer suffered pure economic loss. However, it is not correct to say that the advisor did not owe a duty of care in respect of such loss, as explained above. (C) is not correct. On these facts, it is true that the advisor owed a duty of care to the consumer. However, this choice does not give the correct reason. The duty does not arise simply because the advisor was in business as an investment advisor. Rather, it arises because of a special relationship involving an assumption of responsibility by the advisor and reasonable reliance by the consumer. (E) is not correct. It is true that it must be reasonable for the consumer to rely on the advice. On the facts, it does appear reasonable for the consumer to have relied on the advice without further independent enquiry. Her savings were modest, she could not afford to pay for advice, and the advisor offered a free consultation knowing the purpose of the advice and that it would be relied on. Therefore, a duty of care is owed.

How well did you know this?
1
Not at all
2
3
4
5
Perfectly
11
Q

A worker at a petrochemical plant was severely burned when a pipe carrying hot oil exploded. The worker brought an action against the company that manufactured the pipe. At trial, the worker presented evidence that the pipe burst because it had corroded at a higher than normal rate, and that this was caused by small fissures formed in the pipe coating during manufacturing. The manufacturer presented evidence that the risk of such fissures was known but could not be entirely eliminated despite its rigorous checking process, which met the highest industry standards.

Which of the following statements best describes whether the worker will succeed in a claim against the manufacturer?

The claim is not likely to succeed because the manufacturer could not have discovered the defect in the pipe even with the exercise of reasonable care, so the ‘state of the art’ defence in the Consumer Protection Act 1987 will apply.

The claim is likely to succeed because the manufacturer is liable in negligence because the pipe was defective and injured the worker.

The claim is not likely to succeed because the manufacturer did not fall below a reasonable standard of care and so is not liable in negligence.

The claim is likely to succeed because the fact that the pipe was defective means that the manufacturer is liable under the Consumer Protection Act 1987.

The claim is not likely to succeed because the defective pipe was being used in the course of a business, so the Consumer Protection Act 1987 does not apply.

A

(D) The claim is likely to succeed because the manufacturer is liable to the worker under the Consumer Protection Act 1987 (‘CPA’). The manufacturer is a potential defendant under the CPA as producer of the pipe. The pipe is defective because its safety was not such as persons generally are entitled to expect. The manufacturer is liable to the worker because he suffered damage caused by the defect in the product. Liability under the CPA is strict; it is not necessary to prove that the defect was caused by any fault on the part of the manufacturer. On the facts, there is no defence to the claim under the CPA, as discussed below. (A) is not correct because the ‘state of the art’ defence under the CPA will not apply. This defence applies where the producer could not have discovered the defect because of the state of scientific and technical knowledge at the time. However, this defence has been narrowly interpreted. It does not apply where the risk of a defect is known about, even if the knowledge does not exist to enable it to be avoided. (B) is not correct because it does not state the correct reason why the claim is likely to succeed. The manufacturer is not likely to be liable in negligence. A duty of care was owed to the worker, but the manufacturer does not appear to have fallen below a reasonable standard of care, so is not in breach of duty. (C) is not correct. It is true that the manufacturer is not likely to be liable in negligence because it did not fall below a reasonable standard of care. However, the claim is likely to succeed under the CPA, as explained above. (E) is not correct. The CPA does apply and the claim is likely to succeed, as explained above. The CPA does not apply to damage caused to property used for business, but that is not the case here. The CPA does apply where damage is caused by a defective product, even if the product is used for business purposes, as is the pipe in the question

How well did you know this?
1
Not at all
2
3
4
5
Perfectly
12
Q

A customer takes their suit to a dry cleaners for cleaning. On the wall behind the dry cleaners’ counter is a prominent notice which says ‘All cleaning is subject to our standard terms and conditions, a copy of which is available on request’. The customer leaves their suit with the dry cleaners and is given a ticket, but does not ask to see the terms and conditions. The terms and conditions say that the dry cleaners exclude all liability for any loss of or damage to items left for cleaning, however caused.

Is the exclusion of liability likely to be incorporated into the contract between the customer and the dry cleaners?

Yes, because sufficient steps were taken to bring the exclusion to the customer’s attention.

No, because the terms and conditions were brought to the customer’s attention too late.

Yes, because merchants are free to exclude liability so long as the fact that the contract is subject to terms is clearly displayed.

No, because the customer did not ask for a copy of the terms and conditions.

No, because the terms of the exclusion needed to be drawn specifically to the customer’s attention.

A

(E) It is possible to incorporate terms, including exclusions of liability, into a contract by referring to them in a notice. However, the more onerous or unusual the term, the greater the effort required to bring it to the other party’s attention. This is a particularly broad exclusion of liability covering, for example, damage caused by the dry cleaner’s negligence. It is unlikely to be incorporated into the contract unless the exclusion itself (and not just the fact that all cleaning is subject to terms and conditions) has been drawn specifically to the customer’s attention. Therefore, (A) is incorrect. (B) is not correct because the contract was concluded at the latest when the suit was accepted for cleaning and the customer was handed the ticket, and the customer should have seen the notice before then. (C) is incorrect because it is too broad. As explained above, exclusion is possible, but onerous exclusions require more specific notice. (D) is incorrect because a notice such as this is capable of incorporating terms and conditions into a contract even if they have not been read.

How well did you know this?
1
Not at all
2
3
4
5
Perfectly
13
Q

A junior solicitor is supervising a large team of paralegals, all of whom provide legal services in the personal injury department of a law firm. The solicitor’s supervision extends to managing issues such as requests for holiday leave and other employment-related matters.

Which of the following statements best describes what the SRA expects in terms of supervising non-solicitor employees?

The SRA does not have any expectations about supervision of non-admitted staff.

The SRA expects supervising solicitors to remain accountable for the work carried out by non-solicitor staff.

The SRA expects owners of the firm, not individual solicitors employed in it, to ensure the supervision of non-solicitor staff.

The SRA expects solicitors to supervise non-solicitor staff in employment-related matters only.

Supervision is not a regulatory concern.

A

(B) In acting as a supervisor or manager to others providing legal services, a solicitor must remain accountable for the work carried out through them and effectively supervise work being done for clients. The owners of an SRA-authorised firm must ensure that there is effective supervision. This is justified because of the need to ensure that the firm is well managed and a safe place from which to deliver services to clients. (A) is incorrect because the SRA’s expectation is that all staff in an SRA-authorised firm (whether legally qualified or not) are subject to effective supervision. (C) is incorrect because both firms and individual solicitors have supervisory responsibilities. (D) is incorrect because the SRA requires supervision of the provision of legal services and not just employment-related matters. (E) is incorrect because regulatory expectations as to supervision are contained in the SRA Standards and Regulations.

How well did you know this?
1
Not at all
2
3
4
5
Perfectly
14
Q

A client is contemplating suing her former solicitor for negligent advice in respect of her divorce settlement. The client contends that she received significantly less than her legal entitlement because of her solicitor’s negligent failure to seek disclosure of her former husband’s assets and negotiate a proper settlement.

The client first consulted the solicitor on 10 May 2018, received written advice recommending settlement on 1 August 2018, and the settlement was concluded on 1 September 2018.

Which of the following best describes the applicable limitation period?

Three years starting from the date of the first consultation.

Six years starting from the date of the first consultation.

Three years starting from the date of the advice recommending settlement.

Three years starting from the date the settlement was concluded.

Six years starting from the date the settlement was concluded.

A

(E) The limitation period for a non-personal injury (‘PI’) negligence-based claim is six years. The time starts to run from the accrual date, which is the date the tortious act occurred. Here, that would be the date at which the negligent settlement took place. (A), (C), and (D) are incorrect as the three-year period applies only to PI negligence-based claims. (A) and (B) are also incorrect because there was no negligence at the point of the initial consultation. (C) is further incorrect because the advice does not cause injury until the settlement is finalised.

How well did you know this?
1
Not at all
2
3
4
5
Perfectly
15
Q

A manufacturer failed to pay a supplier for goods the manufacturer ordered, claiming the goods were not delivered. The supplier did not want to jeopardise its relationship with the manufacturer, who, at the time, was the supplier’s biggest customer. However, the relationship between the two has soured and the manufacturer seldom places orders with the supplier. The supplier decides that it will make a claim for the debt. However, the end of the limitation period is near. It ends on 20 November. On 5 November, the supplier’s solicitors issue court proceedings to stop the limitation clock running.

Which of the following best describes the deadline for service of the claim form upon the manufacturer?

The claimant must serve the claim form before 20 November.

The claimant must serve the claim form within 14 days of issue, that is, by 19 November.

The claimant must serve the claim form within 14 days of 20 November, that is, by 4 December.

The claimant must serve the claim form within four months of issue, that is, by 5 March.

The claimant must serve the claim form within one month of issue, that is, by 5 December

A

(D) A claimant must serve an issued claim form within four months of the date of issue, failing which the court will not permit the claimant to continue with the claim. It will be extremely difficult to persuade the court to allow the claim to continue if the claimant fails to serve in time. (A) is incorrect because the claimant need only issue the claim form, not serve it, by the end of the limitation period. (B) is incorrect because the period is four months, not 14 days. Fourteen days refers to the period that the claimant has available to serve Particulars of Claim following service of the claim form. (C) is incorrect for the same reason - it refers to the period for serving the Particulars of Claim. Additionally, the trigger date is not 20 November, which represents the end of the limitation period. (E) is incorrect because the period is four months, not one month.

How well did you know this?
1
Not at all
2
3
4
5
Perfectly
16
Q

A client is contemplating entering into a conditional fee agreement with a solicitor to fund a professional negligence action against another solicitor. Additionally, the client wishes to enter into an after the event insurance policy.

Which of the following best describes what the after the event policy will cover?

The solicitors’ costs (own solicitor and other side) whether or not the client is successful.

The solicitors’ costs (own solicitor and other side) and disbursements whether or not the client is successful.

The solicitors’ costs (own solicitor and other side) and any damages payable to the other side.

The disbursements and the other side’s solicitor’s costs in the event the claim is unsuccessful.

The disbursements and the other side’s solicitor’s costs in the event the claim is successful.

A

(D) If the claim is unsuccessful, the after the event insurance policy will cover disbursements and the other side’s solicitor’s costs. Disbursements are expenses incurred by the solicitor to progress the claim, such as experts’ reports, court fees, medical records, and so on. After the event policies also cover the risk of having to pay costs to the other side if the client’s claim is unsuccessful. (A) and (B) are incorrect as the policy will only cover liability to pay the other side’s costs and the client’s own disbursements. After the event insurance is often associated with a conditional fee agreement, so the client will likely not have any liability to pay his own solicitor’s costs if unsuccessful, but this would be due to the conditional fee agreement rather than the after the event insurance policy. (C) is incorrect as an after the event policy does not cover payment of damages. (E) is incorrect because after the event insurance does not apply when the claim is successful. In that case, disbursements and costs are recovered from the defendant.

How well did you know this?
1
Not at all
2
3
4
5
Perfectly
17
Q

A firm intends to carry out a marketing campaign in which it will advertise that it will share 25% of its fees with anyone who refers personal injury clients to them.

Which of the following best describes whether the arrangement is permissible?

The arrangement is permissible because the share of fees paid by the firm will be for marketing purposes.

The arrangement is not permissible because it will result in the payment of prohibited referral fees.

The arrangement will only be permissible if the firm has an agreement in writing with anyone who refers clients to the firm, and if the firm notifies each referred client of the arrangement.

The arrangement is not permissible because referral fees are prohibited in all matters.

The arrangement will only be permissible if each referred client consents to the arrangement.

A

(B) The Legal Aid, Sentencing and Punishment of Offenders Act 2012 (LASPO) prohibits the payment or receipt of referral fees in claims for damages following personal injury or death. Because the firm will share its fees in exchange for referrals of personal injury matters, this arrangement would result in the payment of prohibited referral fees. (A) is incorrect because the fee paid will be in exchange for the referral, not for marketing; therefore, it will be a prohibited referral fee. (C) is incorrect. This choice states the general rule for permitted fee sharing arrangements, but this is a prohibited referral fee because it is a personal injury matter. (D) is incorrect because payments in exchange for a referral are generally permitted, but are prohibited in personal injury matters. (E) is incorrect because this is a prohibited referral fee, and the client’s consent will not solve the problem.

How well did you know this?
1
Not at all
2
3
4
5
Perfectly
18
Q

The police learn that an animal rights group intends to hold a protest outside a scientific laboratory that conducts testing on animals. The area where the group intends on protesting is restricted to only employees of the laboratory. The laboratory is located in central England, and around 40 activists are likely to attend. A previous protest two months ago caused damage to the windows of the laboratory, and some activists have stated that they “want to finish the job this time”.

Which of the following best explains what the police can do in advance of the protest?

Because the protest is a trespassory assembly, the police can apply to the Home Secretary to ban it.

Because the protest is a trespassory assembly, the police can apply to the local council to ban it.

Because the protest is a public assembly, the police can apply to the Home Secretary to ban it.

Because the protest is a public assembly, the police can apply to the local council to ban it.

Because the protest is a trespassory assembly, the police must direct the protestors to conduct the protest in a manner so as to prevent disorder or damage to property.

A

(B) Because the protest is a trespassory assembly, the police can apply to the local council to ban it before it takes place. A trespassory assembly includes any assembly of 20 or more persons on land to which the public have no right of access, which is likely to be held without the permission of the occupier, and which may cause significant damage to buildings of scientific importance. The planned protest here meets this definition. Because the protest is to take place outside of London, the application to ban the assembly has to be made to the local council rather than the Home Secretary. (A) is incorrect because the Home Secretary can ban trespassory assemblies only in London and Greater London. (C) and (D) are incorrect because the protest does not qualify as a public assembly, as the public have no right of access to the land. (E) is incorrect because the police have the ability to ban a trespassory assembly before it occurs. They are not required (nor are they likely) to allow it to proceed.

How well did you know this?
1
Not at all
2
3
4
5
Perfectly
19
Q

A cyclist suffers multiple injuries following a collision with a car. The solicitors for the cyclist write to the driver’s insurers indicating that they are intending to bring a claim. They subsequently send a letter of claim to the driver, with a copy to his insurers, stating that they will commence formal legal proceedings unless they receive a formal admission of liability within 21 days.

The driver’s insurers respond on Day 20, stating that they are investigating the claim and will come back to the cyclist’s solicitors as soon as possible with confirmation of whether liability can be admitted. The cyclist instructs his solicitors to ignore this and issue proceedings, having already obtained a medical report.

What is the likely impact of the cyclist’s pre-action conduct?

The cyclist is in breach of the personal injury protocol, and he is likely to be penalised in costs or interest.

The court will not consider the conduct of the cyclist to be unreasonable if the claim is subsequently settled.

The court will debar the cyclist from relying upon the medical report obtained prior to commencement.

The driver will be entitled to apply to strike out the claim.

The court will discount any award of compensation by 10% to take into account the unreasonable nature of the cyclist’s pre-action conduct.

A

(A) The cyclist is in breach of the Personal Injury Protocol because the protocol provides that if a defendant responds to a letter of claim, the claimant must then give the defendant three months to investigate the claim. As the claimant did not comply, the judge is likely to be unhappy at the pre-action conduct of the claimant and his solicitors, and the judge may deprive the cyclist of interest on his damages. The judge also may order the cyclist to pay part of the driver’s wasted costs because the claim may have settled earlier without significant cost of court proceedings, if the cyclist had complied with the protocol. (B) is incorrect because the court is very likely to find the conduct of the cyclist unreasonable. (C) is incorrect because the rules do not provide for denial of relying on a medical report as a penalty for non-compliance with the protocol. (D) is incorrect because failure to accord with protocol does not automatically give the other party a right to judgment or to strike out the claim. (E) is incorrect because the rules do not permit the court to reduce the damages to reflect non-compliance with protocol.

How well did you know this?
1
Not at all
2
3
4
5
Perfectly
20
Q

The current government lacks a majority of seats in the House of Commons. This leads the Commons to pass a motion of no confidence in the government.
Which of the following best explains what happens next under the Fixed-term Parliaments Act 2011?

The Prime Minister must immediately resign.

The Queen must immediately dismiss the Prime Minister.

A general election must be held if the House of Commons has not passed a motion signaling confidence in the government within 14 days of the no confidence motion

A general election must be held immediately.

A general election must be held if, within seven days of the no confidence motion, the House of Commons has not passed a motion signaling confidence in the government.

A

(C) A general election must be held if the House of Commons does not pass a motion of confidence in the government within 14 days of the no confidence motion. Normally, general elections in the House of Commons are held every five years. However, under the Fixed-term Parliaments Act 2011, an earlier election will be held if the Commons passes a motion of ‘no confidence’ in the government and that is not followed within 14 days by a motion of confidence in the government. (A) and (B) are incorrect because those events do not necessarily follow a no confidence vote. (D) is incorrect because an election does not immediately follow the no confidence vote. Rather, the 14-day period must expire without a vote of confidence in the government. (E) is incorrect because the period is 14 days, rather than seven days.

How well did you know this?
1
Not at all
2
3
4
5
Perfectly
21
Q

A claimant in a personal injury case issues proceedings after the defendant inadvertently failed to reply to the letter of claim. Upon receiving the Particulars of Claim, the defendant’s solicitor is considering how to respond. Because there was no response to the letter of claim, there has been no opportunity to investigate.

The solicitor is aware that there are numerous witnesses to the accident, all of whom he will need to interview and probably an attendance at the accident scene will be necessary. It will be impossible to complete all of this investigatory work and file a defence within the standard timescale for filing a defence.

Which of the following describes the best steps for the defendant’s solicitor to take now in addition to acknowledging service?

File a defence as best they can within 14 days of the acknowledgment.

Request the claimant to allow them an additional 14 days (42 days in total from service of the Particulars) in which to file a defence.

Request the claimant to allow him an additional 28 days (56 days in total from service of the Particulars) in which to file a defence.

Make an application to the court for an additional 28 days (56 days in total from service of the Particulars) in which to file a defence.

The solicitor should file the acknowledgment of service and file a blanket defence within 14 days of the acknowledgment.

A

(C) A defendant must respond to a claim within 14 days of the deemed date of service of the Particulars of Claim. One possible response is an acknowledgment of service with an indication they will file a defence, which ordinarily must then be filed within 14 days of the acknowledgment (a maximum of 28 days from the deemed date of service of the Particulars). However, the defendant may ask the claimant to agree an extension of up to another 28 days (a total of 56 days from the deemed date of service of the Particulars). If the claimant refuses the extension, or it becomes clear that even more time is needed, the defendant can apply to the court for additional time. Under our facts, the solicitor believes they cannot complete their investigation within the 28-day period. Therefore, the best next step for the solicitor is to ask the claimant for an extension. (A) would not be a good approach because the solicitor will not be able to complete the investigation within 14 days and so will be unable to file a full and detailed defence. (B) is possible, but as the solicitor may ask for an additional 28 days rather than just 14, that would be more prudent. (D) is incorrect because the defendant should first ask the claimant for an extension and should make an application only if the claimant does not agree or if it appears the agreed time will not be sufficient. (E) is incorrect because blanket defences generally are inappropriate. The defendant must admit or deny the claims in the Particulars paragraph by paragraph, and it appears the solicitor would not be able to do so properly without more time for investigation.

22
Q

An artist is selling his work in an art catalogue. His advert states that he will only sell to reputable galleries in London. A dealer calls the artist to purchase one of the paintings and assures him that he is the owner of a well-known gallery in Bond Street, London. The artist agrees to sell him the painting and the contract is agreed orally and the painting collected. A few weeks later the artist’s friend tells him he has seen his work on display in a small gallery in a Northern seaside town.

Does the artist have any recourse against the dealer under the law of mistake?

Yes, the artist would be able to argue ‘non est factum’ as he was entering into a contract that was radically different to what he had envisaged.

Yes, the location of the gallery was fundamental to the contract, so it is likely that the contract would be held to be void.

No, the artist’s only remedy is to rescind the contract for misrepresentation.

No, the location of the gallery was not fundamental to the contract, so the artist’s only remedy is to sue the dealer for breach of warranty.

No, the location of the gallery was not fundamental to the contract, so the artist would have no recourse against the dealer.

A

(B) When one party is mistaken as to the identity of the other, the contract may be void, provided that the identity of the party is of fundamental importance to the other party. It is clear from the scenario that this is the case for the artist, so (B) is the correct answer. (A) is incorrect because non est factum specifically relates to written contracts, and means that a party is bound by their signature unless they mistakenly believe that the document signed represents something completely different from what it actually represents. There must be a fundamental and radical difference, and the signatory must not have been careless in signing the document. (C) is incorrect because, while the contract may be voidable for misrepresentation if it is not void for mistake, this is not the artist’s only remedy. The difference is important, because a mistake makes the contract void, not voidable. (D) is incorrect because the location of the gallery was of fundamental importance to the artist, and because there is no suggestion that the dealer’s assurance has become a term of the contract. (E) is incorrect for the same reason, and also because it may be the case that the artist would have a claim for misrepresentation rather than having no recourse to the dealer.

23
Q

The owner of a hardware store in Newbury has a contractual dispute with a lawn mower importer from Southampton. Both parties have incorporated their businesses and have properly registered their companies at Companies House. Early on, the importer directed the store owner to contact him only through his nominated solicitors. After negotiations break down, the store owner commences proceedings against the importer. To this end, the store owner issues a claim and mails a sealed copy of the proceedings to the importer’s place of business, which is the company’s registered office.
Was the importer validly served?

Yes, because service by first class post to the importer’s registered office is the only valid method of service here as the importer is a company.

No, because service on the importer’s nominated solicitors is the only valid method of service here as the importer is a company.

Yes, because service either by first class post to the importer’s registered office or on the nominated solicitors was proper under the circumstances.

No, because personal service on the importer is the only valid method of service here under the circumstances.

Yes, because service either by first class post to the importer’s registered office or personal service on the importer was proper under the circumstances.

A

(C) Generally, when a defendant has nominated solicitors to accept service, the claim must be served on the nominated solicitors. However, if the defendant is a company, service by delivery or posting to the company’s registered agent will also be sufficient. Here, the defendant is a company and service was made by post to the defendant’s registered office. (A) and (B) are incorrect because, as just explained, service here is not limited to either first class post or delivery to the defendant’s nominated solicitors, as either one will do. (D) and (E) are incorrect as personal service would not be a proper method of service because the defendant is a registered company and has nominated solicitors to receive process.

24
Q

A claimant in a breach of contract case issues proceedings on 1 March. However, there is a delay in serving the defendant, and the claim form has a deemed service date of 5 May. The claim form is marked “Particulars of Claim to follow”. The claimant serves the Particulars of Claim on 16 May.

Was the Particulars of Claim served within the prescribed timescale?

Yes, as a claimant must serve the Particulars of Claim within four months of service of the issuance of proceedings.
No, as a claimant must serve the Particulars of Claim along with the claim form.
Yes, as a claimant must serve the Particulars of Claim within 28 days of service of the claim form.
No, as the claimant must serve the Particulars of Claim within two months of the issuance of proceedings.
Yes, as a claimant must serve the Particulars of Claim within 14 days of service of the claim form.

A

(E) Following service of the claim form, a claimant has 14 days to serve the Particulars of Claim. However, in any case, service must take place within four months of the issuance of proceedings. Here, the claim form was served on 5 May and the Particulars of Claim was served on 16 May, which is within the 14-day prescribed period. Additionally, the proceedings were issued on 1 March, and 16 May is well within the four-month outer limit. (A) is a true statement, but the limiting factor under the facts here would be the date of the service of the claim form, as the end of the 14-day period comes long before the end of the four-month period under our facts. (B) is incorrect. Whilst the Particulars of Claim may be included with the claim form, as explained above, the Particulars may be served later. (C) and (D) are incorrect as they do not state valid timescales.

25
Q

A parent is acting as a Litigation Friend for his 17-year-old son, who was injured in a road traffic accident whilst riding through a busy street on his scooter. The Litigation Friend is ready to issue proceedings as the defendant with whom his son collided continues to dispute responsibility for the accident.

Which of the following documents must the claimant enclose in this type of case when sending the claim form to the court for issue?

A statement signed by the Litigation Friend. This should deal with the Litigation Friend’s fitness to act.
A signed form confirming that the minor is happy for the Litigation Friend to act.

A letter from the defendant (or their representatives) making it clear that they have no objections to the Litigation Friend acting in this case.

A certificate signed by the minor’s family doctor with an assessment of the minor’s ability to understand the litigation process and the purpose of the proceedings.

A ‘certificate of suitability’, which should certify the Litigation Friend’s suitability to act on behalf of the claimant.

A

(E) The claimant must file a certificate of suitability, without which the court will be unable to issue the claim form. In the certificate, the Litigation Friend certifies their suitability to act on behalf of the claimant and confirm that they will be responsible for the costs of the claim. (A) is incorrect because there is no requirement for a witness statement from the Litigation Friend. The certificate of suitability provides this. (B) is incorrect because the minor lacks the capacity to give authority, hence the requirement for a Litigation Friend. (C) is incorrect because the defendant has no influence over the appointment of a Litigation Friend on behalf of the claimant and has no say as to whether they are suitable. This is a matter for the court. (D) is incorrect because the court assumes that a person under the age of 18 lacks capacity. No doctor’s report is necessary, and in the case of very young children would clearly be unnecessary.

26
Q

A woman died last month. In her valid will she appointed her husband as her executor, but her husband has since died. The will leaves £25,000 to the woman’s cousin and the remainder of the woman’s estate to her brother and sister, in equal shares.

Which of the following describes who will best be entitled to take a grant of representation, and which type of grant is appropriate?

The brother and sister are best entitled to take a grant of probate.

The brother, sister, and cousin are equally entitled to take a grant of administration.

The cousin has the best right to take a grant of representation.

The woman’s mother, who is not a beneficiary of the will, is best entitled to take a grant of representation with will annexed.

The brother and sister are entitled to take a grant of letters of administration with will annexed.

A

E) The brother and sister are entitled to take a grant of letters of administration with will annexed. As the woman left a will, rule 20 of the Non-Contentious Probate Rules (‘NCPR’) will apply. Under rule 20, the executor named in the will has the best right to apply for a grant of letters. If the executor has not survived or has renounced, the person(s) with the next best claim under rule 20 are the trustees of the residuary estate and then other residuary beneficiaries (here, the brother and sister). When a testator leaves a will naming an executor and the executor is unable to act, the application for a grant of representation is for a grant of letters of administration with will annexed. (A) is incorrect as only the executor named in a will can apply for a grant of probate. (B) and (C) are incorrect as the cousin’s entitlement (as a beneficiary of a non-residuary gift under the will) is lower than the residuary beneficiaries’ entitlement. Additionally, as explained above, the appropriate grant would be letters of administration with will annexed. (D) is incorrect as the woman’s mother is not entitled to apply for the grant as she does not fall into any of the categories of applicant under NCPR rule 20. Also, there is no such grant as a “grant of representation with will annexed.”

27
Q

A man two months ago, leaving a will in which his brother was appointed as his sole executor. The man owed £25,000 to a known creditor, who the executor was initially unable to trace. The executor, therefore, distributed the estate, but the creditor has now made contact and is seeking payment of the £25,000 from the executor.

How could the executor have best protected himself against liability for a future claim from the creditor?

The executor should have obtained an indemnity from the residuary beneficiaries.

The executor should have placed advertisements in the London Gazette and waited two months from the date of the man’s death before distributing the estate.

The executor should have obtained a Benjamin Order.

The executor should have placed appropriate advertisements in the London Gazette and waited at least two months from the date of the advertisements before distributing the estate.

The executor should have waited six months from the issue of the grant of probate before distributing the estate.

A

(C) Seeking a Benjamin Order is the only way for the executor to gain full protection from a future claim by a known creditor. A Benjamin Order is a court order giving the personal representative (PR) leave to distribute the estate taking into account a possible claim from a person who cannot be found. (A) is incorrect. An indemnity provides some protection to the PR, but there is a risk that the beneficiaries might have insufficient funds to cover the indemnity payment. Consequently, this is not the best protection for the executor. (B) and (D) are incorrect. Placing advertisements will not protect against a claim from a known creditor. (E) is incorrect. Individuals have six months to make financial provision claims, but there is no such time limit for known creditors.

28
Q

A woman died recently. Her will leaves her house to her son and the remainder of her estate to her sister. The woman’s son would prefer for his own adult daughter to receive the house.

Which of the following is the best method for the woman’s son to achieve this outcome?

The son should disclaim his right to the house.

The son should make a written variation passing the house to his daughter.

The son should execute a new will, leaving the house to his daughter.

The son should make a lifetime gift of the house to his daughter once he inherits the house.

The son should tell the executors to transfer the house to his daughter instead.

A

(B) The son should make a written variation passing the house to his daughter. A variation allows a beneficiary to change who receives their inheritance. To be effective for tax purposes, a variation must be made in writing, made within two years of death, and not made for monetary consideration. (A) is incorrect. When a beneficiary disclaims a gift, the gift falls into the residue. So here, a disclaimer would not pass the house to the son’s daughter, and instead the house would pass to the woman’s sister as part of the residuary estate. (C) is incorrect because the will would not become effective until the son’s death, and he wants his daughter to receive the house now. (D) is incorrect. A lifetime gift would not be the best method because it would be a potentially exempt transfer, which would become taxable if the son were to die within seven years. (E) is incorrect. For the variation to be read back to the date of death as if the deceased had left the asset to the new beneficiary, the variation must be in writing. A verbal variation is not effective

29
Q

Two executors are administering the estate of a woman who died recently. The woman had a daughter, currently aged 15, who has an interest in the estate conditional upon her reaching the age of 18. The executors will continue to hold the daughter’s interest (comprising a sum in a high interest bank account) on trust in the meantime. The will contains no provisions in relation to the holding of this sum.
The daughter has asked for the interest generated on the bank account to be paid to her now.

Which of the following best states the position of the executors in relation to this request?

The executors must accumulate the interest received in the bank account and cannot pay it to the daughter until she is 18.

The executors are required to pay the interest to the daughter now that she has requested it.

The executors have full discretion to pay the interest to the daughter if they wish.

The executors can choose to apply the interest only for purposes related to the daughter’s maintenance, education, or benefit.

The executors must pay the interest to the daughter if she is able to demonstrate a need for this money.

A

(D) The executors can choose to the apply the interest only for purposes related to the daughter’s maintenance, education, or benefit. This situation concerns the executors’ power to maintain a minor. When property is held for a minor beneficiary, PRs may apply the income for the maintenance, education, or benefit of the minor. Otherwise, the PRs must accumulate the income. When the daughter turns 18, she will receive any accumulated income. (A) is incorrect. The executors have a discretion to apply income for the daughter’s maintenance, education, or benefit before she reaches the age of 18. (B) is incorrect. There is no requirement for the executors to pay any income to the daughter if she requests it. (C) is incorrect. The executors can choose to apply income for the daughter’s benefit but only for the specified purposes of her maintenance, education, or benefit. Also, the executors cannot pay the interest to the daughter directly because she is a minor. (E) is incorrect. Even if the daughter can demonstrate a need for the money, the executors have a discretion to continue to accumulate interest rather than applying the income for her maintenance.

30
Q

A man’s will leaves £10,000 to a beneficiary. There is very little cash within the estate, but the man owned a large quantity of valuable antiques, which are not left to any specific beneficiary. The man’s executors wish for the beneficiary to accept antiques worth £10,000 instead of the cash gift. The will contains no provisions on appropriation.

Which of the following statements best sets out the legal position in relation to this situation?

The beneficiary has no choice and must accept the antiques instead of the cash gift.

The executors have no power to offer an alternative gift to the beneficiary.

The beneficiary can choose to accept the antiques instead of the cash gift.

If the beneficiary refuses to accept the antiques, he will lose all entitlement under the will.

The executors can offer alternative gifts only of assets equivalent to cash, such as quoted shares.

A

(C) The beneficiary can choose to accept the antiques instead of the cash gift. The executors have the power of appropriation. This means that they can use an alternative asset to satisfy a legacy or interest in the estate, provided no specific beneficiary is affected. Unless the will provides otherwise, the beneficiary must consent to the appropriation. Here the beneficiary can choose whether or not to accept the antiques. (A) is, therefore, incorrect. The beneficiary can decide not to accept the antiques. (B) is incorrect. The executors’ power of appropriation allows them to use an alternative asset to satisfy a gift. (D) is incorrect. The beneficiary can refuse to accept the antiques but will retain his entitlement to £10,000. (E) is incorrect. Provided no specific beneficiary is affected, any asset can be used to satisfy a pecuniary legacy in this situation.

31
Q

An elderly woman died last year, leaving an estate worth £750,000 to various friends and relatives in her will. The woman’s will appoints her partner as the executor and the main beneficiary. However, the woman’s children are concerned about the validity of the will, as the woman had been diagnosed with dementia before making the will, and it includes some unexpected gifts.

What should the woman’s children do to prevent her partner from obtaining a grant of probate whilst the validity of the will is further investigated?

Lodge a caveat at the Probate Registry.

Apply to court for a judge to determine the validity of the will.

Seek a citation to take probate.

Seek a citation to accept or refuse a grant.

Try to resolve matters in the first six months after the woman’s death, as no grant can be issued during this time

A

(A) The children’s best course of action is to lodge a caveat with the Probate Registry to prevent the issue of a grant of representation. This is an appropriate course of action in cases such as this, where there are concerns over the validity of the will. Caveats are valid for up to six months, and if the matter cannot be resolved, it may end up with a judge determining the issue. (B) is incorrect. Although a judge may eventually need to determine the validity of the will, this would not prevent the woman’s partner from obtaining a grant of probate in the meantime. (C) and (D) are incorrect. Citations do not prevent the issue of a grant. Instead, they are used to try to progress the issue of a grant. (E) is incorrect. There is no such limitation on when a grant can be issued.

32
Q

An executor is administering the estate of a man who died recently. There are sufficient assets to pay off the man’s debts, which comprise a mortgage of £50,000 and £10,000 of unsecured debts. The will contains no provisions relating to the payment of debts and makes the following gifts:

A house (worth £100,000 and subject to a £50,000 mortgage) to the man’s son;

A valuable watch (worth £50,000) to the man’s brother;

Quoted shares (worth £50,000) to the man’s daughter; and

The residuary estate (worth £100,000) to the man’s wife.

How should payment of the man’s debts be funded from the estate?

The executor should pay the mortgage and other debts from the residuary estate.

The executor should pay the mortgage from the sale of the house and sell the watch to pay the other debts.

The executor should sell the house to pay off the mortgage and other debts.

The executor should pay the mortgage from the sale of the house and pay the other debts from the residuary estate.

The executor should use funds generated from the house, watch, shares, and residuary estate to pay off all debts on a proportionate basis.

A

(D) The executor should pay the mortgage from the sale of the house and pay the other debts from the residuary estate. This situation involves the payment of debts of a solvent estate, as there are sufficient assets to meet all liabilities. A secured debt, such as the mortgage on the house, should be paid from the asset against which it is secured. This means that the house will be sold to pay off the mortgage. Unsecured debts should be paid from the residuary estate, rather than from specific legacies, such as the gifts of the watch and shares. Property specifically gifted in the will should not be used to pay debts unless other assets have been exhausted. Accordingly, (A), (B), (C), and (E) are incorrect.

33
Q

An executor is dealing with the estate of a man who died recently. The estate is large and is subject to an initial inheritance tax payment of £250,000. The estate comprises a house worth £500,000, £200,000 in various bank accounts (all of which participate in the Her Majesty’s Revenue and Customs direct payment scheme), £200,000 of unlisted shares, a car worth £10,000, and some antiques worth £20,000. One of the beneficiaries has said she is able to offer a loan to fund the inheritance tax that cannot be funded from elsewhere.

Which of the following assets will need to be used to fund the initial payment of inheritance tax?

£200,000 in bank accounts, the car, the antiques, and a £20,000 loan from the beneficiary.

£200,000 of shares, the car, and a £40,000 loan from the beneficiary.

The house only.

£200,000 in bank accounts and £50,000 of the shares.

The car, the antiques, and a £220,000 loan from the beneficiary.

A

(A) The bank accounts, car, antiques, and a £20,000 loan from the beneficiary will need to be used to pay the inheritance tax (‘IHT’). The initial payment of IHT must be made before the PRs can apply for the grant. Consequently, the house and unlisted shares will be unavailable to fund the IHT, as these assets can be sold only with a grant as evidence of the executor’s authority to sell these assets. However, the £200,000 in bank accounts can be paid directly to HMRC before the grant is obtained to fund the IHT. The car and antiques can also be sold to raise funds before the grant is obtained. This leaves an outstanding balance of £20,000, which can be funded via a loan from the beneficiary. (B), (C), (D), and (E) are therefore incorrect.

34
Q

A man is the sole registered proprietor of the home which he lives in with his wife. The spouses’ marriage has broken down and the wife is seeking legal advice because she is concerned that her husband might sell the house.

How should the wife protect her position?

The wife does not need to take any steps as her occupation confers an overriding interest which will bind a buyer.

The wife must register a Class F land charge.

The wife must register a notice on the property register of the title.

The wife must register a home rights notice on the charges register of the title.

The wife must put a Form A restriction on the register to protect her beneficial interest in the property.

A

(D) The wife must register a home rights notice on the charges register of the title. The right of occupation enjoyed by a non-owning spouse or civil partner is not capable of being an overriding interest. Thus, that spouse or civil partner will register their home right as a notice on the registered proprietor’s register of title. (A) is incorrect because, as explained above, the wife’s right to occupy the matrimonial home as a non-owning spouse specifically does not confer an overriding interest. (B) is incorrect because this is how the wife would protect her interest if title were unregistered. Here, title is registered (given the fact that the man is indicated as the ‘sole registered proprietor’). (C) is incorrect because the notice would appear on the charges register. The property register denotes the address, whether the property is freehold or leasehold, and any rights benefitting the property. (E) is incorrect because a Form A restriction is not relevant here. A Form A restriction may be entered on the proprietorship register to denote a person’s beneficial interest in the property. The wife’s relevant interest here arises from her statutory right of occupation of the home as a spouse.

35
Q

A bank lent money to a company which is now in liquidation. The loan was secured by a correctly registered fixed charge over the company’s warehouse premises. The company owes £800,000 on the loan, and the premises are sold for £700,000. The liquidator has £250,000 in other funds remaining after paying the costs of liquidation and the preferential creditors. The total unsecured debt is £2.5 million.

How much will the bank receive?
£800,000
£770,000
£710,000
£700,000
£10,000
A

C) The bank has a fixed charge over the premises, which means the bank is entitled to the proceeds of the sale up to the amount of the debt still owed. Since the proceeds of the sale (£700,000) are less than the amount still owed (£800,000), the bank will receive the entire proceeds from the sale of the warehouse. The bank is an ordinary, unsecured creditor with respect to the £100,000 debt still remaining. Each unsecured creditor will receive a share of the other liquidation funds in the proportion that the debt owed to them bears to the whole unsecured debt. The total debt is £2.5 million and the funds available are £250,000. Therefore, the liquidator has one-tenth of the funds necessary to pay all of the unsecured creditors and each will receive one-tenth of what is owing to them. The bank’s unsecured debt is £100,000 and they will receive one-tenth of that. Thus, in addition to the £700,000 from the sale of the warehouse, the bank will receive £10,000 (one-tenth of the £100,000). (A) is incorrect. Even though the liquidator has sufficient funds to pay the whole debt left owing to the bank, as explained above, the bank will receive only a proportionate share of the unsecured amount. (B) is incorrect as the bank is not entitled to one-tenth of the secured debt in addition to the proceeds of sale. (D) is wrong as although the premises raise £700,000, the bank is owed £800,000 and, therefore, the bank ranks as an unsecured creditor for the outstanding balance. (E) is incorrect as this is the sum the bank will receive in respect of the balance not covered by the sale of the premises; the bank will also receive the sale proceeds of the premises.

36
Q

An appropriately qualified company secretary is appointed by a construction company pursuant to the provisions of the Companies Act 2006. In their first month in office, the secretary enters into five contracts without express authority from the board. The board refuses to honour the obligations imposed under all five of the contracts, arguing that the secretary lacked authority to bind the company in each case.

In which of the following cases is the company’s argument most likely to be successful?

The secretary entered into a lease of a new business premises for the company.

The secretary entered into an expensive long-term contract for maintenance of the office water fountains.

The secretary entered into a contract for the hire of a photocopier, which the company secretary used for their own non-business-related purposes.

The secretary entered into a contract with Companies House to electronically file company documentation.

The secretary entered into a contract to hire cars for the business.

A

A) Company secretaries do not have any statutory powers but are generally able to enter into contracts of an administrative (as opposed to a commercial) nature. A company taking on new business premises would be a commercial decision and, therefore, such a contract would be outside the authority of a company secretary. (B) is incorrect because a contract to maintain the company’s fountain is of an administrative nature, so it would be within the scope of a company secretary’s authority. (C) is incorrect as even though the company secretary has used the photocopier for their own purposes, a contract for the hire of a photocopier by a construction company would fall within the usual authority of a company secretary. A third party would expect a company secretary to have the authority to enter into a contract of this nature, so the secretary would have apparent authority to enter into the contract. The fact that the secretary actually used the photocopier for personal purposes does not destroy the secretary’s apparent authority. (D) is incorrect because part of the role of a company secretary is to make necessary filings at Companies House, and a contract dealing with this would fall within the category of administration. (E) is incorrect because hiring cars would again be classed as a contract of an administrative nature.

37
Q

A bank has agreed to lend a buyer £100,000 for the purchase of a property with registered title. The bank and the buyer will enter into a legal charge.

How will the banks interest be protected?

By a notice on the charges register.
By a notice on the property register.
By a restriction on the proprietorship register.
By a restriction on the charges register.
By a notice on the proprietorship register.

A

(A) The legal charge the parties will enter is a mortgage. A mortgage is a burden on the land as it gives the mortgagee the right to repossess and sell the land in the event the borrower defaults. For registered property, it is protected by a notice on the charges register. (B) is incorrect as the property register gives details of the property and any rights which benefit the property. (C) and (E) are incorrect as the proprietorship register details who holds the legal title to the property. (C) (and (D) as well) is also incorrect in that a restriction is used to prevent any dealing with the land otherwise than in accordance with the registered restriction. A notice does just that – notifies the world of the interest.

38
Q

A company has adopted the Companies (Model Articles) Regulations 2008 (unamended) as its articles of association. It has four directors who are all equal shareholders. One of the directors wishes to retire from the company.

What will happen to the director’s shares when they retire?

On the director’s retirement, the other shareholders must buy the retiring director’s shares.

The shares are automatically retired and the company must return the director’s paid-in capital.

A director who resigns must transfer their shares to the remaining shareholders.

The director may transfer their shares to a third party, and the board must register the transfer.

The director may transfer their shares to a third party, but the board have a discretion to refuse to register the transfer.

A

(E) On retirement, a director can keep their shares or transfer them to someone else. However, the Model Articles give the directors an absolute discretion to refuse to register a transfer of shares. (A) is incorrect as in the Model Articles there is no requirement for the other shareholders to buy a retiring director’s shares. Neither do the articles include a rule that the shares must be retired and the director’s paid-in capital returned, or requiring the remaining shareholders to purchase the retiring director’s shares, so (B) and (C) are incorrect. (D) is incorrect because the board has an absolute discretion to refuse to register the transfer under the Model Articles.

39
Q

A landlord and tenant entered into a lease of shop premises on 1 January 1994. The term of the lease is 35 years. The original tenant has long since left the premises and there have been five assignments of the lease since it was entered. The current tenant has run into financial difficulties and has not paid their rent for five months. The original tenant was very surprised to receive a letter from the landlord’s solicitors demanding payment of the rent arrears and has visited their solicitor for legal advice.

What advice is the solicitor likely to give in relation to the claim by the landlord?

As the original tenant has assigned their interest in the lease, they are released from all future liability regarding rent arrears of subsequent tenants.

As the tenant is an original party to the lease, the landlord can pursue them for the rent arrears of the current tenant.

The tenant’s rights depend on whether the landlord and tenant entered an authorised guarantee agreement when they entered the lease.

The tenant need not worry as the landlord’s only remedy against the tenant would be to retain any rent deposit made by the tenant at the beginning of the lease.

Even though the tenant is an original party to the lease, the landlord cannot pursue them for the rent arrears of the current tenant.

A

(B) The key to this question is the date of this lease. It is dated before 1996, it is an old lease and the provisions of the Landlord and Tenant (Covenants) Act 1995 do not apply. Under the privity of contract rules, the original tenant can be pursued by the landlord for rent arrears of subsequent tenants. Thus, (A) is incorrect. (C) is incorrect, as authorised guarantee agreements are applicable only to new leases entered after 1995. (D) is incorrect, because the landlord may pursue the original tenant as explained above. Additionally, if the rent deposit would have been returned to the tenant on assignment; it is not intended to guarantee the performance of subsequent tenants. (E) is incorrect, as the privity of contract rules discussed above apply.

40
Q

A landowner sold part of a field last year. The transfer to the buyer did not contain any provisions relating to rights. The landlord has taken a shortcut across the field to the public highway since the sale completed. The new owner of the field has told the landowner to cease using the shortcut and to use his existing driveway to access the public highway. The landowner does not want to do this because his existing driveway takes a very long route to the public highway. The landowner has visited their solicitor for some advice.

What advice will the solicitor give their landowner client?

The landowner is not entitled to use the shortcut because the landowner did not grant himself a right-of-way over the field when he transferred ownership.

The landowner is entitled to use the right-of-way over the field because it is a right protected by virtue s62 Law of Property Act 1925.

The landowner is not entitled to use the right-of-way over the field, as he did not reserve the right to do so when he sold that part of his field.

The landowner is entitled to use the right-of-way over the field because he has an implied easement of necessity.

The landowner is entitled to use the right-of-way over the field because he has an implied easement under Wheeldon v Burrows [1879].

Easement can be granted impliedly or by prescription but the landowner must have reserved the right.

A

The landowner is not entitled to use the right-of-way over the field, as he did not reserve the right to do so when he sold that part of his field.

41
Q

A company was incorporated several years ago with the Companies (Model Articles) Regulations 2008 (unamended) as its articles of association. The company is currently valued at £5 million and wishes to borrow £1 million to acquire new premises. The loan will be secured by a fixed charge over the premises.

Which of the following best describes the procedure necessary to borrow the money?

The board may resolve to enter the loan and grant the charge without shareholder approval.

The board may resolve to enter the loan and grant the charge only after the shareholders have approved the terms of the loan.

The shareholders may resolve to enter the loan and grant the charge without board approval.

The board may resolve to enter the loan only after the shareholders resolve to grant the charge.

The finance director has the power to enter the loan and grant the charge without approval of the board or the shareholders.

A

(A) Unless there are any restrictions in the articles (unamended Model Articles do not include any), the board has the power to borrow money. Therefore, a board resolution is all that will be needed to enter the loan and grant the charge. No shareholder approval is needed. Therefore, (B), (C), and (D) are incorrect. (E) is wrong because we do not have facts to support it. Decisions can be delegated to an individual director by the board, but nothing in the facts indicates that the board delegated its power here.

42
Q

A woman has exchanged contracts regarding the sale of her property. The contract included a fittings and contents form upon which the woman indicated that she was removing the ornamental shrubs and two small trees from the garden at the property. On the day of completion, the buyer attends the property and tells the woman that she is not entitled to remove the shrubs and trees from the property.
Is the buyer correct?

Yes, because the trees and shrubs form part of the land and must be left at the property on completion.

No, the woman can remove the shrubs and trees because the fittings and contents form forms part of the contract.

Yes, because the trees and shrubs are personal property and can be removed on completion.

No, because the property must be left in the same condition as it was on exchange of contracts.

Yes, due to the degree of annexation, the trees and shrubs form part of the land.

A

(B) Although the trees and shrubs do form part of the land, the woman has indicated on the fittings and contents form that they will be removed, and as this forms part of the contract, she is entitled to take the specified items. (A) is incorrect because the fittings and contents form supersedes the basic legal position. (C) is incorrect because the trees and shrubs are not personal property, they form part of the land. (D) is incorrect because the fittings and contents form permits the woman to remove the trees and shrubs. (E) is incorrect. The degree of annexation is one of the tests to be applied if it is not possible to distinguish between a fixture and a fitting whereas trees and shrubs are, by definition, part of the land.

43
Q

A solicitor is advising a client who is selling part of some farmland which the client owns. The client wants a right of way over the land he is selling.

Which of the following states the legal position with respect to the right of way?

Easements implied by existing use may continue by reservation to the seller.

Easements may be reserved impliedly to the buyer.

The contract for the sale of the land will automatically reserve appropriate rights over the land being sold.

The seller must expressly reserve any easements he wishes to retain over the land he is to sell.

The seller will retain all of the rights he may need by virtue of section 62 of the Law of Property Act 1925.

A

(D) The seller must expressly reserve any easements he wishes to retain over the land he is to sell. A seller should always specifically consider the rights required on a sale and the seller’s solicitor should draft them, rather than seek to rely on implied easements. (A) is incorrect because easements implied by existing use on a sale of part of a landholding are in favour of the land being acquired (not the land being retained by the seller). (B) is incorrect because it is nonsensical (sellers, not buyers, reserve easements). (C) is incorrect because the contract for the sale of land (for example, the widely used Standard Conditions of Sale) does not automatically reserve any such rights in favour of a seller. (E) is incorrect because there can be no implied reservation of easements in favour of the land being retained by the seller on a sale of part by virtue of section 62. Section 62 operates only to grant necessary rights for the benefit of the land being sold off; it does not apply to any reservation of rights needed by the seller

44
Q

A husband is the sole registered proprietor of a matrimonial home he lives in with his wife. The wife contributed a portion of the home’s purchase price. The husband now wishes to take out a mortgage over the property for the purposes of his business. A bank agrees to provide him the mortgage.
Which of the following is a requirement that must be satisfied prior to the bank’s completion of the mortgage?

The wife must be given full financial information.

The husband and wife must choose a solicitor to represent both of them in the process.

The wife must be a registered proprietor of the home for the husband to take out a mortgage over the property.

The wife must be a registered proprietor of the home for the husband to take out a mortgage over the property.

The wife must remove any registered home right notice on the property’s register of title.

A

(A) One requirement that must be satisfied prior to the bank’s completion of the mortgage is that the wife must be given full financial information. If a mortgagee is lending money for a purpose other than the acquisition of the property (for example, to finance one spouse’s business), the other spouse is effectively acting as guarantor of the spouse’s debts. As such, that spouse must be separately represented by another solicitor and given full financial information. These requirements are designed to protect the spouse who is not a registered proprietor of the marital home. Here, since the bank is lending money to finance the husband’s business, the wife must be given full financial information prior to completion of the mortgage. (B) is incorrect because, as explained above, one requirement is that the husband and wife must be represented by separate solicitors. (C) is incorrect because the wife need not be a registered proprietor of the home for the husband to take out a mortgage. The creditor must satisfy itself, though, that she understands the implications of the transaction in a meaningful way. (D) and (E) are incorrect because no such requirements apply in this situation.

45
Q

A man has recently inherited his late mother’s house. The man has confirmed that the title to his mother’s house is unregistered. He cannot locate the title deeds to the house. The man wishes to sell the house and, on the advice of his solicitor, agrees to the making of an application to Her Majesty’s Land Registry (‘HMLR’) prior to the property being marketed.

Which of the following best describes the title that HMLR is likely to award the man in these circumstances?

Title absolute.
Gift title.
Qualified title.
Good leasehold title.
Possessory title.
A

E) It is likely that HMLR will only award a possessory title when presented with the application for first registration based on lost deeds. Possessory title is a class of title based on factual possession of the land rather than documentary evidence. It might be granted, for example, when the title deeds have been lost. Possessory title can later be upgraded to title absolute if the Registered Proprietor can show that the possessory title has not been challenged for 12 years since it was granted. (A) is incorrect because title absolute will not be granted in this circumstance since the title deeds cannot be produced. (B) is incorrect because there is no such class of title. (C) is incorrect because it is not applicable here. Qualified title is rare in practice and is awarded when a specified interest is excepted from the effect of registration and is therefore not covered by the guarantee of title which would otherwise be available. (D) is incorrect because this class of title would be granted if the freehold title is not produced when an application is made to register a leasehold title.

46
Q

An unmarried couple have had an offer accepted to purchase a property which they wish to use as their home. When they viewed the property, the couple met the seller, who is a single woman and the sole registered proprietor. When the couple visited their solicitor to discuss the purchase, they mentioned the fact that they had seen some men’s shoes by the front door and some large coats and suits in a wardrobe. They wondered whether somebody else was living at the property with the seller.

How will the couple’s solicitor discover who is living at the property?

By looking at the Property Information Form and advising their clients to physically inspect the property.

By looking at the Leasehold Information Form and carrying out a local search.

By looking at the Property Information Form and advising their clients to have a survey carried out.

The solicitor will visit the property to establish whether any occupiers are living there.

By looking at the special conditions on the draft contract to see if an occupier has been declared.

A

(A) The seller is required to disclose the presence of any occupiers on the property information form, and the solicitor will advise their buyer client to physically inspect the property again. (B) is incorrect. The leasehold information form does not contain details of occupiers, and the local search is not relevant in this case. (C) is incorrect as the purpose of a survey is to establish the physical condition of the property and not who lives there. (D) is incorrect as a solicitor does not typically visit the property to establish whether there are any occupiers; it typically is up to the buyers to double check what is provided on the property information form regarding occupiers. (E) is incorrect. Once the solicitor has established whether there are any occupiers by looking at the property information form and advising their client to physically inspect, they will then raise an additional enquiry of the seller’s solicitor seeking assurance that the occupier will sign the relevant special condition on the draft contract.

47
Q

A solicitor is acting for the buyer of a residential dwelling. The Property Information Form indicates that the seller converted a single dwelling house into a house with a separate shop at the front, on the first floor. This was ideal for the buyer because she is the sole proprietor of a tailoring business, and the shop is in a good location for the business. The work was carried out six years ago. The solicitor’s local search has revealed no planning permissions relating to the property.

What advice should the solicitor give their client with respect to the conversion?

That the conversion constituted a material change of use and the local authority still has four years to take enforcement action.

That the conversion constituted a material change of use and the local authority still has 10 years to take enforcement action.

That the conversion constituted a material change of use and that the local authority has an unlimited period of time to take enforcement action.

That the conversion constituted a material change of use, but the buyer should not be concerned because the work was carried out by the seller.

That the conversion constituted a material change of use, but the buyer should not be concerned because the local authority can no longer take enforcement action.

A

A) The solicitor should advise the client that the conversion constituted a material change of use and the local authority still has four years to take enforcement action. The time limit for enforcement in relation to a material change of use is 10 years. The question indicates that the work was done six years ago, which leaves a further four years for enforcement action. (B) is incorrect, as the enforcement period is 10 years from the first breach, which happened six years ago. (C) is incorrect, as the local authority has only 10 years to take enforcement action; their remit is not unlimited. (D) is incorrect as planning problems run with the property and not with the property owner. Accordingly, the local authority could pursue the current buyer/owner to correct the problem created by the work carried out by the previous seller. (E) is incorrect because, as explained, on our facts, the local authority still has four years to take enforcement action.

48
Q

The owner of a property with registered title has taken a shortcut to the public highway by walking through their neighbour’s wood land for the last five years. When challenged by the neighbour, the owner went to see their solicitor, because they were convinced that their property enjoyed a right-of-way over the woodland.

How would a right of way be evidenced on the owner’s title?

It would appear as a restriction on the proprietorship register.

It would appear as a notice on the charges register.

It would appear as a restriction on the property register.

It would appear as a notice on the property register.

It would not matter because the owner has acquired right by prescription.

A

(D) A right which benefits a property appears on the property register of the title taking the benefit.

49
Q

A woman is acting as attorney for her elderly mother in the purchase of a brand-new retirement flat. The woman has been to view the property. She believes it is perfect for her mother as it has a large, communal garden and a warden on site. However, the woman is concerned because she thinks that the land upon which the development is being built was formerly used as a petrol station. She telephones the solicitor she instructed for the purchase to express her concern.

What will the solicitor do to address the woman’s concerns?

The solicitor will carry out a local search.

The solicitor will carry out a contaminated land search.

The solicitor will carry out an environmental search.

The solicitor will carry out a drainage and water search.

The solicitor will advise their client to physically inspect the property.

A

C) An environmental search should be carried out on every purchase and will reveal whether the land upon which the property is being built has ever been put to contaminative use. (A) is incorrect. Although a local search will be carried out on every purchase, it will not address the woman’s concerns. The first part of a local search reveals whether there are registrations by the local authority in the land charges register. The second part is a standard set of enquiries of the local authority which include enquiries about roads fronting the property, public rights of way, and planning entries and building regulations relevant to the property. A local search may also include optional enquiries, but none of the enquiries deal with contaminative land use. (B) is incorrect, as there is no search with this name. (D) is incorrect, as drainage and water search reveals only whether the property is connected to public water and foul water drainage services. (E) is incorrect. Physical inspection is considered to be part of the pre-contract searches, but it would not reveal the former use of the land for contaminative uses, as evidence of such use usually is buried in the land itself

50
Q

A solicitor has acted for a limited company, which manufactures furniture, for a number of years, advising on various business matters. The owning shareholders of the company are three siblings. They are also the three directors. Two of the directors work full-time in the business. The third is a non-executive director who is not employed by the company, and does not work in the business. The non-executive director claims that he is being pushed out of the company because the other directors have decided that all profits are to be paid out in salaries to the working directors, and no dividends are to be paid to shareholders. The non-executive director seeks advice.

Which of the following best describes how the solicitor should proceed?

The solicitor can advise the non-executive director because he is a shareholder in the company.

The solicitor can advise the non-executive director because he is a director of the client, namely the limited company.

The solicitor can advise the non-executive director if the other two directors consent.

The solicitor cannot advise the non-executive director because there is a conflict of interest with the company.

The solicitor cannot advise the non-executive director because there is a conflict of interest between the directors.

A

D) The client is the limited company, a separate legal entity. The conflict of interest would be between the non-executive director and the company. (E) could be correct, if all three directors were also clients of the firm, but this does not necessarily follow from acting for the company. (A) and (B) are incorrect because the client is the company. The solicitor should not give advice to the non-executive director where there is a conflict between the non-executive director and the company. (C) is incorrect because neither of the limited exceptions to client conflicts of interest apply.